5
$\begingroup$

Many years ago, I needed a lower bound for a $(1- \alpha )100 \% $ one-sided confidence interval for $| \mu |$, assuming $X\sim N( \mu , \sigma^2I_n) $ with $ \sigma^2 $ known. Now $\bar{X}\sim N( \mu , \sigma^2/n) $. Let $\bar{x} $ denote the sample mean. At the time, I used a graphical approach to justify the argument that the lower bound $ \omega_ \alpha $ for this sample could be found as the solution for $| \mu |$ that satisfies

$$P_{|\mu |}( |\bar{X} |\leq |\bar{x} |) = 1- \alpha \tag 1.$$

That is, set $ \omega_ \alpha $ equal to the $| \mu |$ that satisfies (1). If (1) produces valid lower bounds then it is relatively straightforward (numerically) to find it in this case since the distribution of $| \bar{X} |$ is a folded normal, and the resulting bound seems correct. My problem is that I do not recall the theoretical justification for (1), even though I expect that the justification for (1) is quite elementary. For background, let’s see how (1)’s equivalent works correctly for determining the lower bound for the one-sided confidence interval for $\mu $.

One-sided 95% confidence lower bound for $\mu $ given $ \sigma^2 $ known.

The equivalent formulation to (1) in this case, with $1- \alpha = 0.95$, is

$$P_\mu ( \bar{X} \leq \bar{x} ) = 0.95. \tag 2$$

Now $P_\mu ( \bar{X} \leq \bar{x} )= P_0 (Z \leq \sqrt{n} (\bar{x}- \mu )/ \sigma ) $ and we know $ P_0 (Z \leq 1.645)=0.95$. Thus, from (2), $\sqrt{n} (\bar{x}- \omega_ \alpha)/ \sigma) =1.645$. Therefore $\omega_\alpha = \bar{x} -1.645 \sigma/\sqrt{n} $, which is the standard confidence lower bound for this case.

My question is: Can using (1) to find the lower bound for the one-sided confidence interval for $| \mu |$ be justified?

$\endgroup$

3 Answers 3

5
$\begingroup$

Yes, this technique works for $|\mu|$. The two key points that make it work are that

  1. the distribution of $|\bar{X}|$ depends on $\mu$ only through $|\mu|$, and
  2. the function $|\mu| \mapsto P_{|\mu|}(|\bar{X}| < y)$ is decreasing in $|\mu|$ for every $y > 0$.

To see (1.), notice that, if $Z \sim N(0, 1)$ is a standard Gaussian, \begin{align*} \bar{X}^2 &\stackrel{\mathrm{d}}{=} \frac{\sigma^2}{n} Z^2 + \frac{2\sigma}{\sqrt{n}}Z\mu + \mu^2 \\ &\stackrel{\mathrm{d}}{=} \frac{\sigma^2}{n} Z^2 + \frac{2\sigma}{\sqrt{n}}Z|\mu| + |\mu|^2, \end{align*} where the second equality holds because $Z \stackrel{\mathrm{d}}{=} \operatorname{sign}(\mu) Z$ by the symmetry of $Z$ around $0$.

Next, we show a general fact about inverting one-sided hypothesis tests. Specifically, suppose $\theta$ is a parameter and $T(X)$ a test statistic, the corresponding one-sided level-$\alpha$ test for $H_0\colon \theta = \theta_0$ rejects when $T(X) > t_{\theta_0}$, where $t_{\theta_0}$ solves $P_{\theta_0}(T(X) \leq t_{\theta_0}) = 1 - \alpha$.

The confidence set formed by inverting this test is then all those $\theta$ whose corresponding null hypotheses would not be rejected based on $T(X)$. That is, $$ \operatorname{CI}(x) = \{\theta : T(x) \leq t_\theta\}. $$

Writing $F_\theta(y) = P_\theta(T(X) \leq y)$ and assuming that $T(X)$ has a uniformly continuous distribution, the above then becomes \begin{align*} \operatorname{CI}(x) &= \{\theta : F_\theta(T(x)) \leq F_\theta(t_\theta)\} \\ &= \{\theta : F_\theta(T(x)) \leq 1 - \alpha\}. \end{align*}

If $F_\theta(y)$ is decreasing in $\theta$, the above is a one-sided interval $[\omega_\alpha, \infty)$, where $\omega_\alpha$ solves $$ F_{\omega_\alpha}(T(x)) = 1 - \alpha. $$

Now, in your case, $\theta = |\mu|$ and $T(x) = |\bar{x}|$, so $\omega_\alpha$ solves \begin{align*} 1 - \alpha &= F_{\omega_\alpha}(|\bar{x}|) \\ &= P_{\omega_\alpha}(|\bar{X}| \leq |\bar{x}|) \end{align*} as per your supposition.

It remains to show in detail (2.), but this is a straightforward calculus exercise.

$\endgroup$
1
  • $\begingroup$ Thanks, that looks like the logic! I will give some more time for others, and probably tick tomorrow. (I guess you mean the absolute value of the mean in your point 1.) $\endgroup$ Commented Nov 1 at 6:41
3
$\begingroup$

This is just a comment to the other excellent answer by Damian Pavlyshyn noting that when $|\bar x|$ is too small, the confidence set defined by $\{|\mu|:P_{|\mu|}(|\bar X|\le |\bar x|)\le 1-\alpha\}$ will include 0 since $$ \Phi\left(\frac{|\bar x|-|\mu|}{\sigma/\sqrt{n}}\right)-\Phi\left(\frac{-|\bar x|-|\mu|}{\sigma/\sqrt{n}}\right), $$ being the probability that $\bar X$ falls between $-\bar x$ and $\bar x$, will then always be smaller than $1-\alpha$. Still, the coverage appears to match the nominal level of $1-\alpha$ as indicated by the following simulation.

pfoldnorm <- function(x, mu, sigma) {
  pnorm((x-mu)/sigma) - pnorm((-x-mu)/sigma)
}
lowerbound <- function(x, alpha=0.05) {
  n <- length(x)
  absxbar <- abs(mean(x))
  f <- function(mu) {
    pfoldnorm(absxbar, mu, 1/sqrt(n)) - 1 + alpha
  }
  if (f(0)<=0) 
    return(0)
  else
    uniroot(f, c(0, 100))$root
}
coverage <- NULL
mu <- seq(0, 2, by=.1)
for (i in seq_along(mu)) {
  hits <- replicate(1e+5,{
    x <- rnorm(1, mu[i])
    l <- lowerbound(x, .05)
    l <= abs(mu[i])
  })
  coverage[i] <- mean(hits)
}
plot(mu, coverage)
abline(h=.95, col="green")

Created on 2025-11-01 with reprex v2.1.1

$\endgroup$
3
  • 1
    $\begingroup$ I think that in your equation you should have $|\bar{x}|$ instead of $\bar{x}$ in both normal CDFs. Making this change in your code shows the CI achieving the correct coverage for all values of $\mu$. $\endgroup$ Commented Nov 1 at 17:23
  • 1
    $\begingroup$ @DamianPavlyshyn You're absolutely right, my bad, the coverage indeed appears to exactly match the nominal level! $\endgroup$ Commented Nov 1 at 21:42
  • $\begingroup$ Thanks Jarle for checking that. $\endgroup$ Commented Nov 1 at 22:11
1
$\begingroup$

Thank you, Damian Pavlyshyn for an excellent answer to my question. My ‘answer’ here is just to provide clarifying details on the calculation of confidence interval for $\mu $ to avoid any misunderstandings. As the distribution of $| \bar{X} |$ is a folded normal, we numerically solve for $ \omega_ \alpha $ the following equation: $$ \Phi\left(\frac{|\bar x|-\omega_ \alpha }{\sigma/\sqrt{n}}\right)-\Phi\left(\frac{-|\bar x|-\omega_ \alpha }{\sigma/\sqrt{n}}\right)=1-\alpha, $$ This can be done quite easily, using, for example, Excel Solver. Solutions to this equation are feasible for $|\bar{x} |> z_{\alpha /2} \sigma/\sqrt{n}$, where $z_{\alpha /2} $ defined by $\Phi (z_{\alpha /2} )=1- \alpha /2$. For $|\bar{x} |\leq z_{\alpha /2} \sigma/\sqrt{n}$, the CI lower bound $ \omega_ \alpha =0$.

Aside

As $\bar{x} >0$ increases, $ \omega_ \alpha $ rapidly approaches $\bar{x} -z_{\alpha } \sigma/\sqrt{n} $. This is not surprising since $\max(0, \bar{x} -z_{\alpha } \sigma/\sqrt{n} $ is the tighter $(1- \alpha )\times 100 \% $ one-sided confidence interval for $\mu $ that would have been able to be used had we known that $\mu \geq 0$.

$\endgroup$

Your Answer

By clicking “Post Your Answer”, you agree to our terms of service and acknowledge you have read our privacy policy.

Start asking to get answers

Find the answer to your question by asking.

Ask question

Explore related questions

See similar questions with these tags.